Вы находитесь на странице: 1из 26

CONTROL

SYSTEMS
Chapter 3 : Time Response Analysis
 

 GATE Objective & Numerical Type Solutions


Question 4 [Practice Book] [GATE EC 1991 IIT-Madras : 2 Mark]
A unity feedback control system has the open loop transfer function.
4(1  2 s )
G (s)  2
s ( s  2)
If the input to the system is a unit ramp, the steady state error will be
(A) 0 (B) 0.5 (C) 2 (D) 
Ans. (A)
4 1  2 s 
Sol. Given : G  s   2 , and r (t )  t u (t )
s  s  2
  Taking Laplace transform of r(t), we get
1
R( s)  2
s
Steady state error is given by,
sR( s)
ess  lim sE ( s )  lim  
s 0 s 0 1  G ( s ) H ( s )

1
s 2
s 1
ess  lim  lim 0
s 0 4 (1  2s ) s 0 4 (1  2s )
        1 2 s  
s ( s  2) s ( s  2)
Hence, the correct option is (A).
Alternatively,
Velocity error coefficient is given by,
s  4(1  2 s )
K v  lim sG ( s ) H ( s )  lim 
s 0 s 0 s 2 ( s  2)
Steady state error for ramp input is given by,
1 1
ess   0
Kv 
For type - 2 system steady state error due to ramp input will be zero.

Question 6 [Practice Book] [GATE EE 1991 IIT-Madras : 2 Mark]


A first order system and its response to a unit step input are shown in figure below, the system parameters
a and K are respectively
c(t )

2.0

K
r (t ) c(t )
s+a
t (sec)
0 0.2
(A) 5, 10 (B) 10, 5 (C) 2, 10 (D) 10, 2

  1 
Ans. (A)
C ( s) K
Sol. For first order system loop transfer function is  and its response to a unit step input are shown
R(s) 1  s
in figure below.
c(t )
c(t ) = K (1 - e - t / t )

t (sec)
0 t
K
C ( s)
For the given transfer function  a and its response.
R( s) 1  s
a
c(t )

2.0

K
r (t ) c(t )
s+a
t (sec)
0 0.2
From the figure, time constant is 0.2 sec.
1
On comparing with above standard equation, we get,    a  5.
a
Using final value theorem steady state output can be written as,
css  lim c (t )  lim s C ( s )
t  s 0

K K 1 K
css  lim s R( s )  lim s  
s 0 sa s  0 sa s a
From figure, css  2  
K
                        2  K  10  
a
Hence, the correct option is (A).
Question 7 [Practice Book] [GATE EC 1992 IIT-Delhi : 8 Marks]
Block diagram model of a position control system is shown in figure.
Amplifier Motor
1
R( s) KA Y (s)
s (0.5s + 1)

sK t

(a) In absence of derivative feedback ( K t  0), determine damping ratio of the system for amplifier gain
K A  5. Also find the steady state error to unit ramp input.
(b) Find suitable values of the parameters K A and Kt so that damping ratio of the system is increased to
0.7 without affecting the steady-state error as obtained in part (a).

  2 
Sol. Given : The given block diagram of a positional control system is shown in figure below.
Amplifier Motor
1
R( s) KA Y (s)
s (0.5s + 1)

sK t

(a) Given : K t  0
5 10
G ( s)   and H ( s )  1
s (0.5 s  1) s ( s  2)
Closed-loop transfer function for negative feedback is given by,
10
G( s) s ( s  2) 10
T ( s)    2 ….. (i)
1  G( s) H ( s) 10 s  2 s  10
1
s ( s  2)
Transfer function for second-order system with unit step input is given by,
C ( s) 2n
 2                …..(ii)
R( s) s  2 n s  n2
where,   damping ratio, n  natural angular frequency
On comparing equation(i) and (ii), we get
n  10 rad/sec. and 2  n  2
1

10
Steady state error is given by,
sR( s)
ess  lim sE ( s )  lim
s 0 s 0 1  G ( s) H ( s)  
1
s
s2 1
      ess  lim  lim  0.2           Ans. 
s 0 10 s 0 10
1 s
s ( s  2) ( s  2)
(b) The open loop transfer function can be written as,
2K A 2K A
OLTF  2 
s  2s  2sKt s [ s  (2  2 Kt )]

The closed loop transfer function can be written as,


Y ( s) 2K A
CLTF   2 …..(i)
R( s ) s  s (2  2 Kt )  2 K A
Transfer function for second-order system with unit step input is given by,
C ( s) 2n
                ….. (ii)
R( s) s 2  2 n s  n2
where,   damping ratio, n  natural angular frequency

  3 
Velocity error coefficient is given by,
K v  lim s G ( s ) H ( s )
s 0

2K A 2K A
K v  lim s 
s 0 s [ s  (2  2 Kt )] 2  2 Kt
Steady state error is given by,
1
ess   
Kv
1
0.2 
2K A
2  2 Kt
0.2 K A  1  Kt ….. (iii)
On comparing equation (i) and (ii), we get
n  2 K A
2  2 Kt  2  n
2  2 Kt 1  Kt
 
2n 2K A
1  Kt
0.7 
2K A
1  K t  0.9899 K A  K A ….. (iv)
From equation (iii) and equation (iv), we get
K A  25  and K t  4 . Ans.
Question 8 [Practice Book] [GATE IN 1992 IIT-Delhi : 1 Marks]
For what values of ‘a’ does the system shown in figure have a zero steady state error [i.e., lim E (t ) ] for a
t 

step input?
E(t) s +1
s 2 + 5s + a

1
s+4
(A) a = 0 (B) a = 2 (C) a  4 (D) No value of ‘a’
Ans. (A)
Sol. The given system is shown in figure.
E(t) s +1
s + 5s + a
2

1
s+4
DC gain of feedback H ( s ) is K H
1 1
K H  lim H ( s )  lim 
s 0 s 0 s4 4

  4 
Closed-loop transfer function is given by,
s 1
G ( s) s  5s  a
2
M (s)  
1  G ( s) H ( s) 1  ( s  1)
( s  5s  a ) ( s  4)
2

( s  1) ( s  4) ( s  1) ( s  4)
M (s)   3
( s  5s  a ) ( s  4) ( s  1) s  4 s  5s 2  20 s  as  4a  s  1
2 2

( s  1) ( s  4)
M ( s)  3
s  9 s  (21  a ) s  (4a  1)
2

Steady state error is given by,


 1 
ess  lim sE ( s )  lim R ( s )   M (s)
s 0 s 0
 KH 
 ( s  1) ( s  4) 
ess  lim s   4  3  R( s)
 s  9s  (21  a ) s  (4a  1) 
s 0 2

1
For unit step input R ( s ) 
s
 ( s  1) ( s  4) 
ess  lim  4  3 
 s  9 s  (21  a) s  (4a  1) 
s 0 2

4
0  4
4a  1
4
4
4a  1
16a  4  4
a0
Hence, the correct option is (A).
Question 9 [Practice Book] [GATE IN 1993 IIT-Bombay : 2 Marks]
A unit step is applied at t  0 to a first order system without time delay. The response has a value of 1.264
units at t  10 min and 2 units at steady state. The transfer function of the system is
2 2 1 1
(A) (B) (C) (D)
1  600s 600  s 1  600s 600  s
Ans. (A)
Sol. The first order system step response is given by,
   
t

c(t )  k 1  e    
 
At t  ,   2  k 1  e   
k  2   
  At  10 min,
  t            
     
 10 

1.264  k 1  e    
                      
  10 min  600sec.  
   First           
order transfer function is given by,
  
k 2
G(s)   Ans.
1  s  1  600 s            

  5 
Question 12 [Practice Book] [GATE IN 1994 IIT-Kharagpur : 1 Mark]
81
A unity feedback closed loop second order system has a transfer function and is excited by a
s  0.6s  9
2

step input of 1 unit. The steady state error of the output is


(A) 10 (B) 0.0 (C) 1.0 (D) 0.1
Ans. (B)
9
9 2
81 s  0.6 s  KG ( s)
Sol. Given : T ( s)  2  …..(i)
s  0.6s  9 1  9 1  G ( s)
s 2  0.6 s
Closed-loop transfer function for negative feedback is given by,
G(s)
T ( s)  …..(ii)
1  G ( s)
On comparing equation (i) and (ii), we get
9
G (s)  2
s  0.6 s
Steady state error is given by,
sR ( s )
ess  lim sE ( s )  lim
s 0 s 0 1  G ( s )

1
s
 s 0
9
1 2
s  0.6 s
  Hence, the correct option is (B).
Assignment 11 [Work Book] [GATE EC 1997 IIT-Madras : 5 Marks]
The figure shows the block diagram representation of a control system. The system in block A has an
impulse response hA (t )  e t u(t ). The system in block B has an impulse response hB (t )  e 2t u(t ). The
block ‘k’ is an amplifier by a factor k. For the overall system the input is x(t ) and output y(t ) .
x (t ) A B y (t )

k
Y (s)
(a) Find the transfer function when k  1.
X (s)
(b) Find the impulse response when k  0.
(c) Find the values of k for which the system becomes unstable.
Sol. Given : Impulse response hA (t )  e t u (t )
Taking Laplace transform, we get
1
H A (s) 
s 1
Impulse response hB (t )  e 2t u (t )
Taking Laplace transform, we get
1
H B ( s) 
s2

  6 
(a) k  1 , G (s)  H A (s)  H B (s)
Closed-loop transfer function for negative feedback is given by,
Y (s) G (s)
T (s)  
X (s) 1  G (s) H (s)
1
( s  1) ( s  2) 1
T (s)   2 Ans.
1
1 s  3s  3
( s  1) ( s  2)
(b) k  0
Y (s) 1 A B
H (s)    
X ( s) ( s  1) ( s  2) s  1 s  2
A  1 and B  1
1 1
H ( s)  
s 1 s  2
Taking inverse Laplace transform, we get
Impulse response h(t )  (e  t  e  2t ) u (t ) Ans.
(c) Value of k for unstable system
k
T (s)  2
s  3s  2  k
Characteristic equation s 2  3s  2  k  0
Routh Tabulation :
s2   1  2 + k 
s1   3  0
0
s   2 + k   

For instability 2  k  0
 k   2 system is unstable. Ans.
Question 8 [Work Book] [GATE EC 1999 IIT-Bombay : 2 Marks]
If the closed-loop transfer function T(s) of a unity negative feedback system is given by
an 1s  an
T s  n n 1
s  a1s  .....  an 1s  an
then the steady state error for a unit ramp input is
a a an 1
(A) n (B) n (C) (D) zero
an1 an 2 an 2
Ans. (D)
an 1s  an
Sol. Given : T  s   n 1
and r(t) = t u (t )
s  a1s  .....  an 1s  an
n

an 1s  an
s  a1s n 1  .....  an  2 s 2
n
T (s)  …... (i)
an 1s  an
1 n
s  a1s n 1  .....  an  2 s 2

  7 
Closed-loop transfer function for negative feedback is given by,
G(s)
T ( s)  …... (ii)
1  G ( s) H ( s)
On comparing equation (i) and (ii), we get
an 1s  an
G( s)  n
s  a1s n 1  .....  an 2 s 2
Taking Laplace transform of r(t), we get
1
R( s )  2
s
The steady state error due to a unit ramp input is given by,
1
ess 
Kv
Velocity error coefficient is given by,
s (an1s  an )
Kv  lim s G(s) H (s)  
s 0 s  a1s n1  .....  an2 s 2
n

1 1
So, that ess    0 
Kv 
Hence, the correct option is (D).
Question 31 [Practice Book] [GATE EC 2000 IIT-Kharagpur : 5 Marks]
The block diagram of a feedback system is shown in the figure.
G
Input Output
s ( s + 3)

G>0

V0 (t )
0.1

Step
response

T
t
(a) Find the closed loop transfer function.
(b) Find the minimum value of G for which the step response of the system would exhibit an overshoot,
as shown in figure.
(c) For G equal to twice the minimum value, find the time period T indicated in the figure.
Sol. (a) The given block diagram of a feedback system is shown below.
G
Input Output
s ( s + 3)

G>0

  8 
Closed-loop transfer function for negative feedback is given by,
G
G(s) s ( s  3)
T ( s)  
1  G ( s) 1  G
s ( s  3)
G G
T (s)   2 Ans.
s ( s  3)  G s  3s  G
(b) Given : Maximum peak overshoot is given by,

2
MPO  e 1  0.1
 Damping ratio is   0.6
For the given response transfer function is
G
T (s)  2 …..(i)
s  3s  G
Transfer function for second-order system is given by,
C (s) 2n
              …..(ii)
R ( s ) s 2  2 n s  2n
where,   damping ratio, n  natural angular frequency
On comparing equation (i) and equation (ii), we get
2  n  3 and 2n  G
3
n   G
2
3
G
2  0.6
G  6.25 Ans.
(c) G '  2G  2  6.25  12.5
2n  G '  n  12.5  3.53
2  n  3    0.424
Damped frequency of oscillation is given by,
d  n 1   2
2
 3.53 1  (0.424) 2  3.197
T
T  1.96 sec. Ans.
Question 37 [Practice Book] [GATE IN 2002 IISc-Bangalore : 2 Marks]
The forward path transfer function of an unity feedback system is given by,

G s 
1  5s 1  10 s 1  2 s 
1  s 1  8s 1  20 s 

If e  t  is the error to a unit impulse input the value of the performance index J   e  t  dt is equal to
0

(A) zero (B) infinity (C) – 12 (D) 0.5


Ans. (D)

  9 
Sol.
E (s)
R( s) G (s) C (s)

B( s)
H (s)

E ( s )  R( s )  B( s ) where e(t ) 


LT
E (s)
E ( s)  R( s)  C ( s) H ( s)
E ( s)  R ( s )  E ( s )G ( s) H ( s )
E ( s ) 1  G ( s ) H ( s )   R ( s )
R( s)
E ( s) 
1  G( s) H (s)
Given : r (t )  (t ), R ( s)  1, H ( s )  1
1
Therefore, E ( s) 
(1  5s ) (1  10 s ) (1  2 s )
1
(1  s ) (1  8s ) (1  20 s )
(1  s ) (1  8s ) (1  20 s )
E ( s) 
(1  s ) (1  8s ) (1  20 s )  (1  5s ) (1  10 s ) (1  2 s )
For causal e( t ) :

E ( s )   e(t )e  st dt
0

Given J   e(t ) dt
0

 J  E (s) s0
(1  s ) (1  8s ) (1  20 s )
J  lim
s 0 (1  s ) (1  8s ) (1  20 s )  (1  5s ) (1  10 s ) (1  2 s )
1 1 1
J 0.5 Ans.
(111)  (111)
Question 11 [Work Book] [GATE EC 2003 IIT-Madras : 2 Marks]
C (s) 4
A second-order system has the transfer function  2 with r(t) as the unit-step function,
R(s) s  4s  4
the response c(t ) of the system is represented by
Step Response Step Response
(A) 1.5 (B) 1.0
Amplitude

Amplitude

1
0.5
0.5
0 0
0 2 4 6 0 2 4 6
Time (sec) Time (sec)

  10 
(C) Step Response (D) Step Response
2.0 1.0

1.5

Amplitude

Amplitude
1.0 0.5

0.5
0 0
0 5 10 15 20 25 0 5 10
Time (sec) Time (sec)
Ans. (B)
C (s) 4
Sol. Given :  2          …..(i)
R( s) s  4s  4
Transfer function for second-order system is given by,
C ( s) 2n
              …..(ii)
R ( s ) s 2  2 n s  2n
where,   damping ratio, n  natural angular frequency
2n  4  n  2 rad/sec
2n  4
 1
Since   1 , system is critically damped.
The final value can be calculated using final value theorem,
4 R( s)
css  lim s 2
s  0 s  4s  4

1
4 s
css  lim 2 s 1
s  0 s  4s  4

For 2% tolerance band settling time is given by,


4 4
Ts    2sec
n 2
This mean that the response c(t ) will be settle to its final value after 2 sec.
Hence, the correct option is (B).
Question 13 [Work Book] [GATE IN 2004 IIT-Delhi : 2 Marks]
A certain system exhibited an overshoot of 16% when subjected to an input of 2u (t ) , where u (t ) is a step
input. The damping ratio and decay ratio respectively are
(A) (0.8, 0.0810) (B) (0.5, 0.0256) (C) (1.0, 0.1626) (D) (1.1, 0.0089)
Ans. (B)
Sol. Given : MPO = 16 % [By default 1st peak overshoot]
Percentage MPO is given by,


1 2
% MPO  e  100


12
16  e  100
 
 ln(0.16)
1  2

  11 
1
2 
3.939
  0.5
Damping ratio ()  0.5

Concept of decay ratio :


c(t)
st
1 peak overshoot
nd
cmax 2 peak overshoot

t
0 tp

2nd peak overshoot


Decay ratio  st
1 peak overshoot

The 2nd peak overshoot is given by,


3 

1 2
%MPO(2 )  end
 100
The decay ratio is given by,
3  3 0.5
 
12 1 (0.5)2
e e 0.00433
Decay ratio  
 0.5
  0.027  0.025
  0.16
12 1 (0.5)2
e e
Hence, the correct option is (B).
Question 51 [Practice Book] [GATE EE 2007 IIT-Kanpur : 2 Marks]

RLC circuit shown in figure. For a step input ei , the overshoot in the output e0 will be

R = 10 W L = 1mH

ei C = 10 mF e0

(A) 0 %, since the system is not under damped (B) 5 %


(C) 16 % (D) 48 %
Ans. (C)

  12 
Sol. The given RLC circuit is shown below.
R = 10 W L = 1mH

ei C = 10 mF e0

Transform domain :

R Ls
1
Ei ( s ) E0 ( s )
Cs

 
 1 
 Cs 
E0 ( s )  Ei ( s )  [By VDR]
1 
         
 R  Ls  
 Cs 
Ei ( s )
  E0 ( s)   
RCs  LCs 2  1
E0 ( s ) 1
        
Ei ( s )  2 R 1 
LC  s  s  
 L LC 
  Characteristic equation is given by,
R 1
  s2  s 0      …(i)
L LC
Standard characteristic equation for second order system is given by,
  s 2  2 n s  2n  0       …(ii)

On comparing equation (i) and equation (ii),


1 R
  n  and 2  n   
LC    L

R 1 R LC R C
            
L 2 n L 2 2 L

10 10 10 6
        0.5  
2 110 3
  Maximum peak overshoot is given by,
  0.5
12 1 (0.5) 2
MPO  e e  
      MPO  0.163 or 16.3%  16 %  
  Hence, the correct option is (C).

  13 
Question 52 [Practice Book] [GATE EE 2007 IIT-Kanpur : 2 Marks]

If the above step response is to be observed on a non-storage CRO, then it would be best have the ei as a
(A) step function (B) square wave of 50 Hz
(C) square wave of 300 Hz (D) square wave of 2 kHz
Ans. (C)
Sol. The resonance frequency is given by,
1 1
n    
LC 110 10 10 6
3

    n  10 4 rad/sec  
  Settling time for 2% tolerance band is given by,
4 4
ts    0.8 msec.  
 n 0.5 104
  To observe the transient response, input must be applied atleast upto ts  0.8 msec .
Peak time is given by,
 
tp    0.36 msec  
n 1   2
10 4
1  (0.5) 2
  To observe peak overshoot, the input must be applied atleast upto t p  0.36 msec.
Case 1 : If step input is applied.
ei

t
0

e0

e0ss = 1

ts= 0.8 ms t
0 tp = 0.36 ms

The given CRO is non-storage which means it can’t record the response. So it is not possible to read the
transient response practically as it will appear only for one t s i.e., 0.8 msec. After 0.8 msec CRO only
displays constant value 1.
Therefore, the constant input e1  1 should be applied again and again (i.e., square wave) so that transient
response appears always on CRO screen.
Case 2 : If square wave input is applied.
ei

t
0

  14 
For obtaining transient response,
T
 (t p and ts )  
2
  Option (B) :
For f1  50 Hz  
T1 1
      10 ms  ts  
2 2  50
10 ms  0.8 ms  
  The output of the CRO is constant in between 0.8 ms to 10 ms as shown below,
e0

e0 ss = 1

t (ms)
0 0.8 10

Option (C) :
For f 2  300 Hz  
T2 1
      1.67 ms  ts  
2 2  300
  The output of the CRO is constant in between 0.8 ms to 1.67 ms which is very less as shown below,
e0

e0 ss = 1

t (ms)
0 0.8 1.67

Option (D) :
For f 3  2 kHz  
T3 1
      0.25 ms  ts  
2 2  2 103
  Since, 0.25 ms is very much less than 0.8 ms hence, the output waveform will not appear on the CRO
screen.
T
Hence 300 Hz frequency square wave is the most suitable as  1.67 msec because only for this
2
frequency transient response will always appear on CRO screen.
Therefore, it would be best to have ei as a square wave of 300 Hz.
Hence, the correct option is (C).
Question 53 [Practice Book] [GATE EE 2007 IIT-Kanpur : 2 Marks]
Consider the feedback system shown below which is subjected to a unit step input. The system is stable
and as following parameters K p  4, K i  10 ,   500 rad/sec and   0.7 . The steady state value of Z is

  15 
Ki
1
s
Z
0 w2
Kp
s + 2xws + w2
2

(A) 1 (B) 0.25 (C) 0.1 (D) 0


Ans. (A)
1
Sol. Given : R ( s ) 
s
The given feedback system is shown below.
Ki
1
s
Z
0 R( s) w2
Kp C (s)
E (s) s 2 + 2xws + w2

2n
G ( s)  2
s  2  n s  2n
The equivalent representation can be drawn as shown in figure below.
E (s) æ Ki ö
R (s) ç Kp + s ÷ G (s) C (s)
è ø

Closed-loop transfer function for negative unity feedback is given by,


C ( s) G '( s)
T ( s)  
R( s) 1  G '( s)
 K 
 K p  i  G (s)
C ( s)
 
s 
R( s)  K 
1   K p  i  G(s)
 s 
From the figure, the error signal can be written as,
  Ki  
  K p   G(s) 
E ( s)  R ( s)  C ( s)  R( s ) 1  
s 

 1   K  Ki  G ( s) 
  p  
 s 
 
1  1 
E (s)   
s  Ki 
1 K  G (s) 
  p s  
From the figure,
K K  s 
Z ( s )  i E ( s )  2i  
s s  s  ( sK p  K i ) G ( s ) 

  16 
 
 
Ki  1 
Z ( s) 
s   2n 
 s  ( sK  K )
i  2 
 s  2 n s  n
p 2
  

Ki  s 2  2 n s  2n 
Z ( s)   
s  s  s 2  2  n s  n2   n2 ( sK p  Ki ) 
 
Steady state value can be calculated as,
2n
zss  lim z (t )  lim s Z ( s )  Ki  1
t  s 0 2n  Ki
Hence, the correct option is (A).
Question 63 [Practice Book] [GATE EE 2011 IIT-Madras : 2 Marks]
A two-loop position control system is shown below.
Motor
1
R( s) Y (s)
s ( s + 1)

ks
Tacho-generator

The gain k of the Tacho-generator influences mainly by


(A) peak overshoot.
(B) natural frequency of oscillation.
(C) phase shift of the closed loop transfer function at very low frequency    0 .
(D) phase shift of the closed loop transfer function at very high frequency      .
Ans. (A)
Sol. Given : A two-loop position control system is shown below.
Motor
1
R( s) Y (s)
s ( s + 1)

ks
Tacho-generator

On solving the inner loop, we get


1
s ( s  1) 1
 2
1
ks s  s (1  k )
s ( s  1)
Now, the overall transfer function can be written as,
Y (s) 1
 2         …..(i)
R ( s ) s  (k  1) s  1

  17 
Transfer function for second-order system with unit step input is given by,
C (s) 2n
 2              …..(ii)
R( s) s  2 n s  2n
where,   damping ratio, n  natural angular frequency
On comparing equation (i) and equation (ii), we get
ω n  1 and 2 ξ ωn  k  1
k 1
So ξ …..(iii)
2
Maximum peak overshoot is given by,

12
MPO  e 0  MPO  1, 0   1
Peak overshoots depends on damping factor ξ and ξ is proportional to gain from equation (iii). So, gain
k of the Tacho-generator influences mainly by peak overshoot.
Hence, the correct option is (A).
Question 20 [Work Book] [GATE EC/EE/IN 2013 IIT-Bombay : 2 Marks]
( s ) 10
The open-loop transfer function of a dc motor is given as  . When connected in feedback
Va ( s ) 1  10 s
as shown below, the approximate value of K a that will reduce the time constant of the closed loop system
by one hundred times as compared to that of the open-loop system is
Va ( s )
10
R( s) +– Ka w( s )
1 + 10s

(A) 1 (B) 5 (C) 10 (D) 100


Ans. (C)
1 ( s ) 10
Sol. Given : closed loop  open loop where  represents time constant and 
100 Va ( s ) 1  10 s
Va ( s )
10
R( s) +– Ka w( s )
1 + 10s

C (s) K ( s ) 10
For first order system loop transfer function is  comparing with  open loop  10
R ( s ) 1  s Va ( s ) 1  10 s
Closed-loop transfer function for negative unity feedback is given by,
G(s)
T ( s) 
1  G (s)
 10 
Here G( s)  K a  
 1  10s 
 10 
Ka  
( s )  1  10s   K a 10 10 K a
 
R( s)  10  1  10 s  K a 10 10 s  (10 K a  1)
1  Ka  
 1  10 s 

  18 
Dividing numerator and denominator by 10 K a  1
10 K a
( s ) 10 K a  1

R(s)  10 
1  s
 10 K a  1 
C (s) K ( s )
For first order system loop transfer function is  . On comparing with we get
R ( s ) 1  s R(s)
10
closed loop 
10 K a  1
1
We have closed loop  open loop
100
10 1
 10
10 K a  1 100
10 Ka  1  100
10 Ka  99
K a  9.9  10
Hence, the correct option is (C).
Question 67 [Practice Book] [GATE EC 2015 (Set-03) IIT-Kanpur : 2 Marks]
The position control of a DC servo-motor is given in the figure. The values of the parameters are
KT  1 N-m/A, Ra  1 , La  0.1H, J  5 kg-m 2 , B  1 N-m/(rad/sec) and K b  1 V/(rad/sec).
The steady-state position response (in radians) due to unit impulse disturbance torque Td is ______.
Td ( s )

KT 1 1
Va ( s ) q( s )
Ra + La s Js + B s

Kb

Ans. – 0.5
Sol. Given : KT  1 N-m/A, Ra  1 , La  0.1H, J  5 kg-m 2 , B  1 N-m/(rad/sec) , K b  1 V/(rad/sec)
Td ( s )

KT 1 1
Va ( s ) q( s )
Ra + La s Js + B s

Kb

For unit impulse Td ( s )  1


1
X (s) Js  B

 TD ( s ) 1  1 K b KT
( Js  B) ( Ra  La s )

  19 
X ( s) 1

 TD ( s ) ( Js  B ) ( Ra  La s )  K b KT
 TD ( s )
( s ) 
s  ( Js  B) ( Ra  La s )  K b KT 
1
( s )   TD ( s)  1
s  ( Js  B) ( Ra  La s )  K b KT 
Steady state response can be calculated using final value theorem.
Applying final value theorem,
1
(0)  lim s  ( s )  lim
s 0 s  0 ( Js  B ) ( R  L s )  K K
a a b T

1 1
(0)     0.5
BRa  K b KT 1  1
Hence, the correct answer is – 0.5.
Question 68 [Practice Book] [GATE EC 2016 (Set-02) IISc-Bangalore : 2 Marks]
1
In the feedback system shown below G ( s )  .
( s  2s)
2

The step response of the closed -loop system should have minimum settling time and have no overshoot.
r K G (s) y

The required value of gain K to achieve this is ________.


Ans. 1
1
Sol. Given : G ( s)  2 , G '( s)  KG ( s)
s  2s
The second order closed loop transfer function with negative unity feedback is given by,
K
Y (s) G '( s)
 s  2s  2
2 K
 ……. (i)
R( s) 1  G '( s) 1  K s  2s  K
s 2  2s
Minimum settling time and no overshoot
 1
From equation (i),
n  K
And 2n  2
2 K  2
1
 1
K
K 1
K 1

  20 
 IES Objective Solutions
Question 4 [Practice Book] [IES EE 1992]
Damping factor and un-damped natural frequency for the position control system is given by
K K
(A) 2 KJ , KJ respectively (B) , respectively
2 fJ J
f K J
(C) , respectively (D) , KJ respectively
2 KJ J 2 Kf
Ans. (C)
Sol. Characteristic equation of position control system is given by,
f K
s2  s 0 ….(i)
J J
Standard form of second order characteristic equation is given by,
s 2  2n s  2n  0 ….(ii)
On comparing equation (i) and (ii), we get
f K
2n  2n 
J J
f J K
 n  rad/sec
2J K J
f

2 KJ
Hence, the correct option is (C).
Question 14 [Practice Book] [IES EE 1995]
Consider a system shown in the given figure.
If the system is distributed so that c (0)  1, then c(t ) for a unit step input will be
2
s

(A) 1  t (B) 1  t (C) 1  2t (D) 1  2t


Ans. (C)
C ( s) 2
Sol. Given : 
U ( s) s
2
C ( s)  U ( s)
s
2
C (s)  2
s
Taking inverse Laplace transform, we get
2
c(t )  L1  2   2t
s 
c (t )  2t  c (0)  2t  1
Hence, the correct option is (C).
  21 
Question 22 [Work Book] [IES EC 2003]
K

Unit Impulse Response


0.63K
0.37K

t1 t2 t

K
The unit impulse response of a system having transfer function is shown above. The value of  is:
s
1 1
(A) t1 (B) (C) t2 (D)
t1 t2
Ans. (D)
C (s) K
Sol. Given :  and R( s)  1
R(s) s  
K
C ( s) 
s
Taking inverse Laplace transform, we get
c(t )  Ke t
1
Time constant t 

Time constant is the time at which
c(t )  Ke 1  0.37 K
1
So,   t2 

1

t2
Hence, the correct option is (D).
Question 54 [Practice Book] [IES EE 2005]
Which one of the following is the steady state error of a control system with step error, ramp error and
parabolic error constants K p , K v and K a respectively for the input (1  t 2 )3 u(t ) ?
3 3 3 3 3 3 3 6
(A)  (B)  (C)  (D) 
1  K p 2Ka 1  K p Ka 1  K p Ka 1  K p Ka
Ans. (D)
Sol. Given : r (t )  3(1  t 2 ) u (t )
Taking Laplace transform, we get
3 6
R( s)   3
s s

  22 
Steady state error is given by,
3 6 
s  3 
sR( s )
 lim 
s s 
ess  lim
s 0 1  G ( s ) s 0 1  G ( s )

3 6
ess  lim  lim 2 2
s 0 1  G ( s ) s 0 s  s G ( s )

3 6
ess  
1  lim G ( s ) lim s 2G ( s )
s 0 s 0

3 6
ess  
1  K p Ka
Hence, the correct option is (D).
Question 72 [Practice Book] [IES EE 2007]
K
For a unity feedback control system with forward path transfer function G ( s )  , what is error
s5
transfer function We ( s ) used for determination of error coefficients?
K K s5 K ( s  5)
(A) (B) (C) (D)
s5 s K 5 s K 5 s K 5
Ans. (C)
K
Sol. Given : G ( s )  , H (s)  1
s5
We ( s ) K
R(s) C(s)
– s+5

We ( s ) 1

R( s) 1  G ( s)
We ( s ) 1

R( s) 1  K
s5
We ( s ) s5

R( s) s  5  K
Hence, the correct option is (C).
Question 77 [Practice Book] [IES EE 2008]
In the time domain analysis of feedback control systems which one pair of the following is not correctly
matched?
(A) Under damped : Minimize the effect of nonlinearities
(B) Dominant poles : Transients die out more rapidly
(C) Far away poles to the left half of s–plane : Transients die out more rapidly
(D) A pole near to the left of dominant complex poles and near a zero : Magnitude of transient is small
Ans. (B)
Sol. Dominant Pole : The poles that are close to the imaginary axis in the left-half s-plane give rise to transient
responses that will decay relatively slowly, whereas the poles that are far away from the axis (relative to
the dominant poles) correspond to fast-decaying time responses.
Hence, the correct option is (B).
  23 
Question 89 [Practice Book] [IES EE 2009]
In a fluid flow system two fluids are mixed in appropriate proportion. The concentration at the mixing
point is y(t) and it is reproduced without change, Td seconds later at the monitoring point as b(t). What is
the transfer function between b(t) and y(t)? (Where S is distance between monitoring point and mixing
point)
(A) e Td (B) e Td s (C) e Td s (D) e Td
Ans. (C)
Sol. Given : y(t )  b(t  Td )
Taking Laplace transform, we get
Y ( s)  eTd s B( s)
Y (s)
 eTd s
B( s )
Hence, the correct option is (C).
Question 97 [Practice Book] [IES EC 2011, 2001]
When two identical first order systems have been cascaded non-interactively the unit step response on the
system will be
(A) Over-damped (B) Under-damped (C) Un-damped (D) Critically-damped
Ans. (D)
Sol. Cascading of two first order system with non-interactively
R R

Amplifier
Vi C with C V0
gain = 1

1 1
Vi ( s ) V0 ( s )
1 + RCs 1 + RCs
V0 ( s ) 1
T (s)  
Vi ( s ) (1  sRC ) 2
The pole-zero diagram of above transfer function
jw

2 poles
s
–1/RC
Two repeated poles.
So, system is critical damped (x = 0).
Hence, the correct option is (D).
Question 108 [Practice Book] [IES EC 2012]
Assertion (A) : A second order system subjected to a unit impulse oscillates at its natural frequency.
Reason (R) : Impulse input contains frequencies from   to   .
Codes :
(A) Both A and R are individually true and R is the correct explanation of A.

  24 
(B) Both A and R are individually true but R is not the correct explanation of A.
(C) A is true but R is false.
(D) A is false but R is true.
Ans. (D)
Sol. For   0 , a second order system subjected to a unit impulse oscillates at its natural frequency but it is not
always true.
c(t) Imaginary
jw

Marginal
0 Un-damped 0 t
stable s

The impulse response contains all the frequency components having frequency response as
L [d(t ) ]

Hence, the correct option is (D).


Question 131 [Practice Book] [IES EE 2014]
The dominant poles of a servo-system are located at s  ( 2  j 2) . The damping ratio of the system is
(A) 1 (B) 0.8 (C) 0.707 (D) 0.6
Ans. (C)
Sol. Given : Pole are located at s  2  2 j …. (i)
Poles of second-order transfer function is given by,
s   n  jn 1   2 …. (ii)
On comparing equation (i) and (ii), we get
n 1  2  2
2n (1   2 )  4  2n  2n 2  4 …. (iii)
and n  2
 22n  4 …. (iv)
From equation (iii) and (iv), we get
2n  8
4 1
2  
8 2
  0.707
Hence, the correct option is (C) .
  25 
Question 135 [Practice Book] [IES EE 2014]
A unity feedback second order control system is characterized by the open loop transfer function
K
G (s)  , H (s)  1
s ( Js  B )
J = moment of inertia, B = damping constant and K = system gain
The transient response specification which is not affected by system gain variation is
(A) Peak overshoot (B) Rise time
(C) Settling time (D) Time to peak overshoot
Ans. (C)
K
Sol. Given : G ( s )  and H ( s)  1
s ( Js  B )
Characteristics equation is given by,
1  G(s) H (s)  0
Bs K
Js 2  Bs  K  0     s 2   0 …. (i)
J J
Characteristics equation for standard second-order system is given by,
s 2  2n s  n2  0 …. (ii)
On comparing equation (i) and (ii), we get
K B
n  and 2n 
J J
B 1

2 KJ
Peak overshoot is given by,

12
Mp e
Since  depends on gain K so peak overshoot affected by K.
Rise time is given by,

tr 
d
Sine d depends on gain K so rise time affected by K, settling time is given by,
4 4 8J
ts   
n B B
2J
Settling time is independent of gain K so ts will not be affected by K.
Hence, the correct option is (C).



  26 

Вам также может понравиться